Inhalt

17.4 Beweis des Satzes über die Jordansche Normalform

Beweis von Theorem 17.3

Die Eindeutigkeitsaussage haben wir bereits in Abschnitt 17.1.2 bewiesen. Ist \(f\in \operatorname{End}_K(V)\) gegeben, so zerlegen wir \(V = \bigoplus _{i=1}^r \tilde V_{\lambda _i}\) in die direkte Summe der verallgemeinerten Eigenräume zu den paarweise verschiedenen Eigenwerten \(\lambda _1,\dots , \lambda _r\) von \(f\), siehe Korollar 17.16.

Wir wählen mit Hilfe von Satz 17.22 Basen der \(\tilde V_{\lambda _i}\), so dass der nilpotente Endomorphismus \(f_{|\tilde V_i}-\lambda _i\operatorname{id}_{\tilde V_i}\) von \(\tilde V_i\) durch eine Matrix in Jordanscher Normalform beschrieben wird. Indem wir alle diese Basen zusammensetzen, erhalten wir eine Basis von \(V\), bezüglich derer \(f\) durch eine Matrix in Jordanscher Normalform beschrieben wird.

Bemerkung 17.23 Berechnung der Jordanschen Normalform einer Matrix/eines Endomorphismus

Um die Jordansche Normalform eines trigonalisierbaren Endomorphismus (bzw. einer Matrix) zu finden, genügt es, die Dimensionen \(\dim \operatorname{Ker}(f-\lambda \operatorname{id})^i\) für alle Eigenwerte \(\lambda \) und alle \(i\) zwischen \(1\) und \(\operatorname{mult}_\lambda (\operatorname{minpol}_f)\) zu berechnen, was man mit (mehrfacher …) Anwendung des Gauß-Algorithmus erledigen kann. Daraus findet man, wie der Eindeutigkeitsbeweis zeigt, die Jordansche Normalform. Oft kann man einen Großteil dieser Berechnungen sparen, wenn man zunächst das charakteristische Polynom und das Minimalpolynom berechnet und in Linearfaktoren zerlegt, weil das gewisse Einschränkungen an die Jordansche Normalform mit sich bringt, siehe Satz 17.4.

Ergänzung 17.24 Berechnung einer Jordanbasis

Sei \(f\) ein trigonalisierbarer Endomorphismus eines endlichdimensionalen \(K\)-Vektorraums \(V\). Eine Basis \(\mathscr B\) von \(V\) zu finden, so dass \(M^\mathscr B_\mathscr B(f)\) Jordansche Normalform hat (eine sogenannte Jordanbasis), ist in der Regel wesentlich aufwändiger, als diese Jordansche Normalform zu berechnen. (Wir haben ja bereits gesehen, dass es im allgemeinen Fall eine umfangreiche Rechnung erfordert, um für zueinander konjugierte Matrizen \(A\) und \(B\) eine invertierbare Matrix \(S\) mit \(B=SAS^{-1}\) zu finden.)

Als erstes berechnet man die verallgemeinerten Eigenräume von \(f\). Auch das kann schon rechenintensiv sein, aber im Prinzip ist klar, wie vorzugehen ist. Danach kann man sich auf den Fall beschränken, dass \(V\) ein einziger verallgemeinerter Eigenraum ist, etwa zum Eigenwert \(\lambda \). Ersetzt man \(f\) durch \(f-\lambda \operatorname{id}_V\), so hat man die Aufgabe auf den Fall eines nilpotenten Endomorphismus reduziert.

Im Prinzip könnte man wie im Beweis von Satz 17.22 vorgehen, um die gesuchte Basis zu konstruieren. Um die Berechnung einigermaßen effizient auszuführen, ist es aber besser, die Sache etwas systematischer anzugehen. Das klärt die Situation vielleicht auch zusätzlich auf (allerdings ist die zusätzlich erforderliche »Buchhaltung« etwas lästig, weswegen wir für Satz 17.22 einen kürzeren Beweis gewählt haben).

Es sei also \(f\colon V\to V\) nilpotent, etwa \(f^m=0\), \(f^{m-1}\ne 0\). Wir betrachten die folgende Kette von Untervektorräumen von \(V\):

\[ 0 \subseteq \operatorname{Ker}(f) \subseteq \operatorname{Ker}(f^2)\subseteq \cdots \subseteq \operatorname{Ker}(f^{m-1})\subseteq \operatorname{Ker}(f^m) = V. \]

Wir wählen nun nacheinander

  • ein Komplement \(U_{m-1}\) von \(\operatorname{Ker}(f^{m-1})\) in \(\operatorname{Ker}(f^m)=V\),

  • ein Komplement \(U_{m-2}\) von \(f(U_{m-1})\oplus \operatorname{Ker}(f^{m-2})\) in \(\operatorname{Ker}(f^{m-1})\),

  • ein Komplement \(U_{m-3}\) von \(f^2(U_{m-1})\oplus f(U_{m-2})\oplus \operatorname{Ker}(f^{m-3})\) in \(\operatorname{Ker}(f^{m-2})\),

  • ein Komplement \(U_0\) von \(f^{m-1}(U_{m-1})\oplus f^{m-2}(U_{m-2})\oplus \cdots \oplus f(U_1)\) in \(\operatorname{Ker}(f)\).

Für alle \(i=0,\dots , m-1\) sei \(u^{(i)}_1, \dots , u^{(i)}_{d_i}\) eine Basis von \(U_i\). Dann bilden die Vektoren

\[ f^j(u^{(i)}_k),\quad i=0, \dots , m-1,\ k=1,\dots , d_i,\ j=0, \dots , i, \]

eine Basis \(\mathscr B\) von \(V\), so dass \(M^\mathscr B_\mathscr B(f)\) Jordansche Normalform hat. Dabei ordnen wir die Basisvektoren so an, dass für jedes \(i\) und \(k\) die Vektoren \(u^{(i)}_k\), \(f(u^{(i)}_k)\), …, \(f^i(u^{(i)}_k)\) direkt hintereinander stehen. Sortiert man noch nach \(i\), so kann man zusätzlich erreichen, dass die Jordan-Blöcke der Größe nach geordnet sind.

Dass diese Familie von Vektoren eine Basis bildet, ist mit Blick auf die Konstruktion der \(U_i\) ohne größere Schwierigkeiten einzusehen.

Dass die Summe \(f^j(U_{m-1}) + f^{j-1}(U_{m-2}) + \cdots + f(U_{m-j}) + \operatorname{Ker}(f^{m-j-1})\) in der obigen Konstruktion in \(\operatorname{Ker}(f^{m-j})\) enthalten ist, folgt aus \(f^m=0\) und der Konstruktion der \(U_i\). Es bleibt aber noch zu begründen, dass diese Summe

\[ f^j(U_{m-1}) + f^{j-1}(U_{m-2}) + \cdots + f(U_{m-j}) + \operatorname{Ker}(f^{m-j-1}) \]

in jedem der obigen Schritte eine direkte Summe ist. Dafür wollen wir zum Abschluss ein Argument skizzieren. Wir führen Induktion nach \(j\). Für \(j=1\) ist die Sache klar. Nehmen wir nun an, dass

\[ f^j(u_{m-1}) + \cdots + f(u_{m-j}) + v = 0 \]

ist mit \(j {\gt} 1\), \(u_i\in U_i\), \(v\in \operatorname{Ker}(f^{m-j-1})\). Wir wollen zeigen, dass alle einzelnen Summanden \(=0\) sind. Durch Anwenden von \(f^{m-j-1}\) erhalten wir

\[ f^{m-1}(u_{m-1}) + \cdots + f^{m-j}(u_{m-j}) = 0. \]

Wenn wir zeigen können, dass daraus \(u_i=0\) für alle \(i=m-j,\dots , m-1\) folgt, dann sind wir fertig.

Wir schreiben die obige Gleichung um als

\[ f^{m-j}(f^{j-1}(u_{m-1}) + \cdots + u_{m-j})= f^{m-1}(u_{m-1}) + \cdots + f^{m-j}(u_{m-j}) = 0. \]

Nach Induktionsvoraussetzung ist \(f^{j-1}(U_{m-1}) \oplus f^{j-2}(U_{m-2}) \oplus \cdots \oplus U_{m-j} \oplus \operatorname{Ker}(f^{m-j})\) eine direkte Summe. Dass das Element \(f^{j-1}(u_{m-1}) + \cdots + u_{m-j}\) in \(\operatorname{Ker}(f^{m-j})\) liegt, wie wir hier sehen, impliziert also \(f^{j-1}(u_{m-1}) + \cdots + u_{m-j} = 0\), und damit \(f^{j-1}(u_{m-1}) = \cdots = u_{m-j} = 0\).

Nun gilt \(U_i\cap \operatorname{Ker}(f^i)=0\) nach Konstruktion von \(U_i\), und daher erhalten wir schließlich \(u_{m-1}=\cdots = u_{m-j}=0\).